LSAT 53 – Section 3 – Question 22

You need a full course to see this video. Enroll now and get started in less than a minute.

Target time: 1:28

This is question data from the 7Sage LSAT Scorer. You can score your LSATs, track your results, and analyze your performance with pretty charts and vital statistics - all with a Free Account ← sign up in less than 10 seconds

Question
QuickView
Type Tags Answer
Choices
Curve Question
Difficulty
Psg/Game/S
Difficulty
Explanation
PT53 S3 Q22
+LR
Except +Exc
Resolve reconcile or explain +RRE
A
29%
162
B
58%
166
C
5%
158
D
4%
158
E
5%
161
148
160
172
+Hardest 145.896 +SubsectionMedium

We’ve got an RRE Except question which we can identify from the question stem: Each of the following, if true, helps to resolve the survey’s apparently paradoxical results EXCEPT:

Let’s start by identifying the apparent paradox. The average number of books read annually per capita (i.e. per person) has gone down every year for three years. Bookstores, however, have increased their profits over that time period.

Any hypothetical resolutions? Well what if books now cost more? So people are reading less but the profit margins for bookstores are higher. What if a lot more people now live in the region we are surveying? Maybe the same number of people are reading the same (or more) books annually, but there are simply more non-readers in the population. This would mean the profits might not take a hit, but the average books read annually would decrease because we have added more non-readers to the pool. What if a bunch of bookstores closed over the past three years? So the amount of books that people are reading has gone down, but the business is now consolidated into a few bookstores who are now getting more business than before.

There are a ton of potential ways to reconcile this info. Which is great because this is an “except” question, meaning we are going to be given 4 answer choices that do reconcile the paradox, and only one that doesn’t. Our job is to find the latter.

Let’s move on to the answer choices:

Answer Choice (A) At first glance it is hard to see what this is doing for us. Libraries are purchasing fewer contemporary novels. Doesn’t that mean less books are being purchased, and isn’t that a fact that we already know about? Well, let’s take a step back. We know that fewer books are being read, but bookstores are making more money. This is giving us a reason why a certain segment of the population might be bringing new business to bookstores. It’s reasonable to assume that frequent library goers who prefer contemporary novels may now be forced to buy these books from bookstores where once they would have borrowed them from libraries. Therefore, even though our per capita rate of books read per year might go down, we have a fact that would lead to profits for bookstores increasing. This is reconciling our discrepancy, but remember, this is an except question–so this AC is wrong!

Correct Answer Choice (B) This is correct! It does absolutely nothing for our paradox! Shoplifting is affecting other businesses but not bookstores. That would give us a reason why profits for bookstores might be better compared to other businesses–but does it give us a reason why profits for bookstores would have gone up? We are told that the increase in shoplifting is recent. Therefore, it’s not like shoplifters were ravaging bookstores and shrinking their profit margins until bookstores got their act together recently and installed security systems. No! The scourge of shoplifting has only affected retail recently. Therefore, we don’t have anything here that would lead to an increase in profits. We just have a reason why profits might not be decreasing at the same rate as other businesses that are more affected by shoplifting. But that’s not a paradox that we care about. This is completely irrelevant to our stimulus, and because this question is an except question, it’s the right answer!

Answer Choice (C) This is great! It gives us an alternative revenue source that can account for the increased profits of bookstores, even if people are buying less books! Therefore, it is unfortunately incorrect since this question is an except question!

Answer Choice (D) This is telling us that people are spending more per book than they used to. This might mean that even if they are buying fewer books, bookstores are making higher profits. Since (D) helps resolve the discrepancy in the stimulus, (D) is wrong!

Answer Choice (E) Just like (C), this is giving us an alternative revenue source besides books. So people are reading fewer books, but they’re buying more magazines! Therefore, it’s quite possible that bookstores are making more money. It’s resolving the paradox in the stimulus, and therefore wrong.

 

Take PrepTest

Review Results

Leave a Reply